You are on page 1of 13

Version 042 midterm 03 chiu (56565)

This print-out should have 17 questions.


Multiple-choice questions may continue on
the next column or page find all choices
before answering.
001

10.0 points

4. Ia, IIa, IIIa


5. Ib, IIa, IIIa
6. Ib, IIa, IIIb
7. Ia, IIb, IIIa
8. Ib, IIb, IIIa

A unipolar generator consists of a copper


disk of radius R rotating in a uniform, steady
magnetic field B perpendicular to the disk,
out of the page. (The magnetic field is produced by large coils carrying constant current,
not shown in the diagram.) Sliding contacts
are made at the center (on the axle) and at the
rim of the disk, and the wires are connected
to a voltmeter. If the outer rim travels counterclockwise at a speed v, choose a correct
answer for each of following questions.
The direction of the instantaneous average
velocity of the electrons at any point along
AB
Ia) upward
Ib) downward
The direction of the magnetic force on the
electrons at any point along AB
IIa) towards A
IIb) towards B
The direction of the conventional current
flow
IIIa) into the + terminal of the voltmeter
IIIb) out of the + terminal of the voltmeter
1. Ib, IIb, IIIb
2. Ia, IIb, IIIb
3. Ia, IIa, IIIb correct

Explanation:
Counterclockwise rotation of the disk implies an upward instantaneous velocity at any
point along AB. Hence the answer is Ia.
~ = q~v B
~ is pointing
The magnetic force F
radially inward. Hence, the answer is IIa.
Negative charge is accumulated at the center of the disk A, or the conventional current
flows out from the + terminal. Hence, the
answer is IIIb.
002 10.0 points
Consider the set up shown in the figure where
a solenoid has a steadily increasing magnetic
flux which generates identical induced emfs
for the two cases illustrated.
i
(1)

#2

#1
A

Case 1: Two identical light bulbs are in


series; the electrical power consumed by bulb
1 and bulb 2 is P1 and P2 , respectively.
C
(2)

#2

#1
A

B
F

Case 2: Bulb 2 is shorted by a wire which


is connected between the two points C and F ;
the electrical power consumed by bulb 1 and
bulb 2 is P1 and P2 , respectively.
P
What is the ratio 1 ?
P1

Version 042 midterm 03 chiu (56565)


P1
=0
P1
P
2. 1 = 3
P1
1
P
3. 1 =
P1
4

P
1
4. 1 =
P1
8

P
5. 1 = 1
P1
P1
1
6.
=
P1
3

P
7. 1 = 2
P1
P
8. 1 = 8
P1
1
P
9. 1 =
P1
2

P
10. 1 = 4 correct
P1
Explanation:
Let E and R be the induced emf and resistance of the light bulbs, respectively.
For case 1, since the two bulbs are in series,
the equivalent resistance is simply Req = R +
R = 2 R and the current through the bulbs is
1.

I=

E
.
2R

Thus for case 1, the power consumed by bulb


1 is

2
E
E2
P1 =
.
R=
2R
4R

For case 2, since bulb 2 is shorted, the


current through bulb 1 is now
E
I = .
R

so the power consumed by bulb 1 is


 2
E
E2

P1 =
R=
R
R
and the ratio is
P1
= 4.
P1

003

2
10.0 points

In an experiment designed to measure the


magnitude of a uniform magnetic field, electrons are accelerated from rest through a potential difference of V = 135 Volts. The electrons travel along a curved path because of
the magnetic force exerted on them, and the
radius of the path is measured to be r = 7 cm.
If the magnetic field is perpendicular to the
beam, what is its magnitude?
Note:
The mass of the electron is
9.1 1031 kg and its charge is 1.6 1019 C.
1. 0.000726
2. 0.000435
3. 0.000792
4. 0.000554
5. 0.000971
6. 0.00056
7. 0.00067
8. 0.00135
9. 0.000528
10. 0.000706
Correct answer: 0.00056 T.
Explanation:
By the conservation of energy
K + U = 0
We note that the initial kinetic energy is zero,
1
and the final kinetic energy is Kf = me v 2 .
2
The change in the potential energy is given by
eV .
Kf = eV

Version 042 midterm 03 chiu (56565)


1
me v 2 = eV
2
s
2eV
v =
me
v =

2(1.6 1019 C)(135 V)


(9.1 1031 kg)

v = 6.89003 106 m/s

1. VI:

2. V:

By equating the magnitude of the magnetic


and centripetal forces we have
evB =

eB =

me v 2
r
me v
r

3. IV:

The magnetic field is given by the expression


B =

B =

me v
er

(9.1 1031 kg)(6.89003 106 m/s)


(1.6 1019 C)(7 cm)

4. I:

B = 0.00056 T

004

10.0 points

correct
5. VIII:
A wire loop of radius R carrying a counterclockwise (when viewed from the right) current I is moving to the right along the x-axis
at a speed v. It passes around a second, stationary wire loop of radius a where a R.
Choose the plot that correctly displays the
qualitative behavior of the induced current
I(t) in the stationary loop. On the plots, the
+I axis represents clockwise current, while
the I axis represents counter-clockwise current (when viewed from the right on the +xaxis).

Version 042 midterm 03 chiu (56565)


6. VII:

dBloop
3
2x
dx

2
2
5/2
dt
2 (x + R ) dt
dBloop
3xv
2
dt
(x + R2 )5/2

7. III:

Examining this result, we see that the presence of x in the numerator means our plot
must pass through 0, while to either side of
t = 0 we should expect some polynomiallike behavior. Combined with what we know
about the sign of the current for times t < 0
and t > 0, I is the only correct option.
005

10.0 points

8. II:

Explanation:
I is the correct choice. Before the current
loop passes the wire ring, the flux through
the ring points to the right and is increasing; to resist this change in flux, a clockwise
(positive) current is induced in the ring. After the loop passes the ring, the flux still
points to the right but is decreasing, inducing a counter-clockwise (negative) current in
the ring. To get a sense of the functional form
of the change in flux, note that we are entitled
to use
1
Bloop
(x2 + R2 )3/2
since a R. (Since we are only interested in the qualitative behavior, we can
leave off the constant factors and concern ourselves only with proportionality.) Since the
area of the ring is not changing, dB /dt =
Aring dBloop /dt. Again ignoring the constant
factor Aloop , we calculate dBloop /dt,


dBloop
1
d

dt
dt (x2 + R2 )3/2

Consider the three configurations shown


in the figure. In each case there is a long
solenoid viewed end-on with current flowing
clockwise, where the current is a decreasing function of time: dI(t)/dt < 0. Midway along each solenoid is a piece of neutral wire arrranged as shown in the figure.
Choose the response that correctly identifies the direction of the induced current in
the neutral wires, where CW=clockwise and
CCW=counter-clockwise. You may assume
the magnetic field outside the interior of each
solenoid 0.
1. I: A:CW, B:no current, C:CW correct
2. VI: A:CW, B:CW, C:CW
3. V: A:CW, B:CW, C:CCW

Version 042 midterm 03 chiu (56565)


4. IV: A:CCW, B:no current, C:CW
5. VII: A:CCW, B:CCW, C:CW
6. VIII: A:CCW, B:CCW, C:CCW

depending on which pole of the magnet is


present inside the cube.
D: B = 0 according to Gausss law of
magnetism.
E: Either C or D can be true depending on
situation.

7. III: A:CCW, B:no current, C:CCW

1. E

8. II: A:CW, B:no current, C:CCW

2. A

9. IX: A:CW, B:no current, C:no current

3. D correct

10. X: A:CCW, B:no current, C:no current

4. B

Explanation:
I is correct. In each case, the solenoid
produces a magnetic field in the shaded region
that points into the page but is decreasing.
The induced current opposes the change in
flux, therefore the neutral loops in A and C
acquire a CW current. In B the loop is not
closed, so no current flows.

5. C

006

10.0 points

A bar magnet (the shaded cuboid object in


the figure) is placed as shown in the figure,
with its axis horizontal and about half of its
length inside the cubical surface of side a.
What can
Z you say about the magnetic flux
~ A
~ through the cubical surface
B =
B.d

shown in the figure? (Note: One half of the


magnet is shaded lighter only to show that it
is inside the cube).
A: B > 0, because there is only one pole
of the magnet inside the surface.
B: B < 0, because there is only one pole
of the magnet inside the surface.
C: B can be either positive or negative,

Explanation:
D is correct. Gausss law of magnetism
holds here. Suppose the North pole of the
magnet was inside. The field lines due to
this pole point outwards at every point on the
surface of the cube outside of the magnet, but
they all come back inside the surface through
the magnet, thus making the net flux zero. If
the south pole is inside, things work the other
way around, with field lines pointing inwards
everywhere on the surface outside the magnet,
and going out through the magnet. In either
case, the net flux through the surface is zero.
007 (part 1 of 2) 10.0 points
Assume: The mobile charge carriers are either
electrons or holes. The holes have the same
magnitude of charge as the electrons.
Note: In the figure, the point at the upper
edge P1 and at the lower edge P2 have the
same x coordinate.
A constant magnitude magnetic field points
out of the paper. There is a steady flow of a
horizontal current flowing from left to right in
the x direction.

Version 042 midterm 03 chiu (56565)


y
x
a

6
~
~vd B

~
B

P1

~
B

~vd

~
B

P2

V
~ = e~vd B
~
F

A voltmeter (with an internal resistance


less than infinity) is connected to the system,
where the contact points are on the upper and
lower surfaces and are in the same vertical
plane.
Choose the correct answer for the case
where the sign of the charge of current carriers are either negative (electrons) or positive
(holes).

The magnetic force on a negative charge


points downwards, so an excess of positive
charge develops along the upper edge. The
induced electric field then points downward,
so P1 is at a higher potential relative to P2 .
The Hall current flows in the direction of the
induced field, so it flows through the voltmeter from top to bottom.

1. The direction of the (conventional) current through the voltmeter is downward for
either positive or negative charge carriers.

+ + + + + + + +

2. The direction of the (conventional) current through the voltmeter is downward for
positive charge carriers and upward for negative charge carriers.
3. The (conventional) current through the
voltmeter is zero for either positive or negative
charge carriers.
4. The direction of the (conventional) current through the voltmeter is upward for positive charge carriers and downward for negative charge carriers. correct
5. The direction of the (conventional) current through the voltmeter is upward for either positive or negative charge carriers.

~
F


008 (part 2 of 2) 10.0 points
Denote the drift velocity by vd and the magnitude of the charge of an electron by q.
Using the dimensions (a, b, and L) given in
the figure, what is the magnitude of the Hall
voltage?
1. VHall = 2 vd B b
2. VHall = 2 vd B a
3. VHall = q vd B b
4. VHall = 2 vd B L

Explanation:

5. VHall = vd B b correct

Version 042 midterm 03 chiu (56565)

The magnitude of the flux is given by


6. VHall = q vd B a
~ A cos
|B | = |B|

7. VHall = vd B L
8. VHall = q vd B L

|B | = (0.0003 T)(0.0314159 m2 ) cos(15 )

9. VHall = vd B a
Explanation:
The force due to the induced electric field
must balance the force due to the magnetic
field, so
q EHall = q vd B .

|B | = 9.10363 106 Wb
010 10.0 points
A very long thick wire of radius R carries a
current I, as in the following figure.

The magnitude of the Hall voltage is


I1 R

VHall = EHall b = vd B b ,
where b is the height of the conducting slab.
009

10.0 points

A square loop of wire of side length s =


28 cm encompasses a circular region of radius
r = 10 cm that contains a magnetic field
~ = 0.0003 T oriented at an angle = 15
|B|
to the perpendicular as shown; outside this
region, B = 0. Calculate the magnitude of
the magnetic flux |B | through the square
loop.
1. 7.17367e-06
2. 6.17275e-06
3. 6.66074e-06
4. 1.12009e-05
5. 9.93678e-06
6. 9.10363e-06
7. 5.79819e-06
8. 8.21603e-06
9. 4.47063e-06
10. 1.67649e-05
Correct answer: 9.10363 106 Wb.

Explanation:
The area of the circular region is given by
A = r 2 = (10 cm)2 = 0.0314159 m2

Find the magnitude of the magnetic field


inside the wire, a distance r from the center
of the wire, where r < R.
Assume current density is the same at every
point inside the wire.
1.
2.
3.
4.
5.
6.
7.
8.
9.
10.

0 Ir
4 R2
0 IR
4 r 2
0 Ir 2
4 R
0 Ir
correct
2 R2
0 IR
2 r 2
0 Ir 2
2 R3
0 Ir 2
2 R
0 IR2
2 r 3
0 IR2
4 r 3
0 Ir 2
4 R3

Explanation:

Version 042 midterm 03 chiu (56565)


The current density (current/area) is the
same throughout the wire. Sketch an Amperian loop within the wire with radius r. Then,
the current through this loop i divided by area
of this loop is equal to the total current per
unit area.
i
I
=
2
r
R2
r2
i=I 2
R
The magnetic field is tangential to the Amperian loop. Apply Amperes law by integrating counterclockwise around the loop if
looking at the loop from the right end. Then
the current flows out of the surface and is
positive.
I
~ d~ = 0 Iinside path
B
r2
R2
0 I r
B=
.
2 R2

B (2 r) = 0 I

011

10.0 points

2. front face, 0.576 mV


3. top face, 0.864 mV
4. left face, 0.684 mV
5. right face, 0.54 mV
6. left face, 0.504 mV
7. bottom face, 0.648 mV
8. top face, 0.9 mV
9. right face, 0.72 mV correct
10. down face, 0.828 mV
Explanation:
Since the box is metallic, it has freely moving conducting electrons. As the cube moves
upward with velocity ~v , electrons inside the
metallic cube will also move with it. Since
they are moving perpendicular to the mag~ they will feel a net magnetic
netic field B,
~ = q(~v B)
~ equation.
force according to the F
The magnetic force will thus push electrons
to the left face creating a net potential difference between the right and left face of the
metal cube, ultimately setting up an electric
field between right and left faces of the solid
cube. This will leave positive charges on the
right face, which will be at a higher potential.
The potential difference across the right
and left faces of the cube is given by
V = Ed

A solid metal cube of edge length d =


1.2 cm moving in the positive Y direction at a
constant velocity v = 6 m/s. The cube moves
through a uniform magnetic field B = 0.01 T
that points in the +z direction.
Which face has the higher potential? Also
find the difference between lower and higher
potential. (Express your answer in units of
mV.)
1. back face, 0.792 mV

When equilibrium for the mobile charges is


established, the electrostatic and magnetic
forces on an electron are balanced.
FE = FB
eE = evB
E = vB
In terms of B, the potential is given by
V = vBd

Version 042 midterm 03 chiu (56565)


V = (6 m/s)(0.01 T)(1.2 cm) = 0.72 mV
012 (part 1 of 2) 10.0 points
Consider the figure shown below. The switch
is initially set at position b. There is no charge
nor current around the right loop while at
position b. At t = 0 the switch is set to
position a.
C

L
E

S b
a
R

What is the current at t = 0?



E
1 e1
R
E
2. I0 =
2R
1. I0 =

3. I0 = 0 correct
4. I0 =

E
R

5. I0 = E R
6. I0 =
EL 1
e
R

EL
8. I0 =
1 e1
R
EL
9. I0 =
R
E
10. I0 = e1
R
Explanation:
The differential equation for an LC circuit
is given by
7. I0 =

dI
= 0,
E IRL
dt
whose solution is

E 
t/L
.
1e
I(t) =
R

Hence at t = 0 we have I = 0. (The back emf


across the inductor achieves its greatest value
at t = 0).
013 (part 2 of 2) 10.0 points
After leaving the switch at position a for a
long time, move the switch from a to b.
There will be current oscillations.
The maximum current will be given by
r
L
1. Imax = E
C
r
C
2. Imax = E
L
E
3. Imax =
correct
R
r
E
L
4. Imax =
R C
r
C
E
5. Imax =
R L

6. Imax = E L C
r
1
7. Imax = E
LC

E
8. Imax =
LC
R
Explanation:
When the switch S is on at a, the loop
equation is given by
E L

dI
IR = 0
dt

After a long time, the current approaches a


dI
= 0. In other words the
constant value, i.e
dt
loop equation becomes
E IR = 0
I =

E
R

Now, the switch S is moved from a to b. The


conservation of the total energy in the LC
circuit implies that at t = 0,
UC + UL = (UL )max =

1 2
LI
2 max

Version 042 midterm 03 chiu (56565)


since UC = 0 at t = 0. Hence, the maximum
current in the LC circuit must be equal to
the current in the circuit at t=0, given by the
expression
E
Imax =
R
014 (part 1 of 2) 10.0 points

In the given figure, +x points to the right,


+y points upwards and +z points out of the
page, towards you. A rectangular current
loop mounted on a pivot aligned along the
z-axis is shown in the figure (since your line
of sight is in the plane of the loop, you can
only see it edge-on, like a straight line). The
loop carries a current I = 1.5 A. A uniform
~ = 6 Tj is present in the
magnetic field of B
region. If the sides of the loop are given as
a = 9 cm (not seen in the figure) and b = 4 cm
respectively, and the angle = 19 degrees,
what is the magnitude of torque acting on the
loop at this instant?
1. 0.004137
2. 0.00293
3. 0.01055
4. 0.002813
5. 0.002433
6. 0.01184
7. 0.00421
8. 0.004002
9. 0.009254
10. 0.004688
Correct answer: 0.01055 Nm.
Explanation:
The area of the loop is given by
A = ab = (9 cm)(4 cm) = 0.0036 m2

10

The magnitude of the torque is given by


= B I A sin
= (6 T)(0.0036 m2 )(1.5 A) sin(19 )
= 0.01055 Nm
One can also obtain the magnitude of the
torque by the following argument.
According to right hand rule, the magnetic
force acting on the near side (the side visible in the figure) and far side of the loop
will be equal and opposite, and act along the
same line in the negative and positive z direction, respectively. Hence, there will not be
any torque due to these forces. The magnetic
force on the left and right sides (the sides
perpendicular to the plane of the figure) will
be along +x and x directions respectively,
and will have the same magnitude F = BIa.
However, their lines of action differ by a distance of d = b sin . Hence, the magnitude of
the torque, which is the moment of the force,
can be obtained.
= F d = BIab sin = BIA sin

015 (part 2 of 2) 10.0 points


For the same loop as in the previous problem,
suppose the magnetic field is a function of
~ = B0 yj.
the y-coordinate, and is given by B
What will be the net force acting on the loop?
Here, A stands for the area of the rectangular loop.
1. B0 IA cos k
2. B0 IA sin k
3. B0 IA sin j
4. B0 IA cos i
5. B0 IA cos j
6. B0 IA sin i correct
Explanation:

Version 042 midterm 03 chiu (56565)


Since we know that magnetic field varies
with the y-coordinate, we expect that the net
force on the loop may no longer be zero. For
any point on the near side, there will be a
corresponding point on the far side with the
same y-coordinate. Hence, the forces on the
near and far sides cancel other exactly. However, this cancellation does not work for the
left and right sides of the loop. All the points
on the left side have the same y-coordinate,
say, y1 , while all points on the right side have
a different y-coordinate, y2 . If B1 and B2 are
the corresponding magnetic fields at y = y1
and y = y2 respectively, then we can calculate
the net force as follows.
~ = B1 Iai B2 Iai
F

~ = (B1 B2 ) Iai
F
~ = (y1 y2 ) B0 Iai
F
~ = (b sin ) B0 Iai
F
~ = B0 Iab sin i
F

016

10.0 points

You pull on a simple loop circuit of dimensions 2d L consisting of an ideal wire and a
resistor. You pull with just enough force so
that the circuit moves at a constant speed v.
Initially, half the circuit lies inside a region
of magnetic field B that is directed out of the
page. How much work must you do to completely remove the circuit from the magnetic
field? Choose the correct answer, expressed
only in terms of the given quantities.
L 2 B 2 d2
1. D:
R

11

LBdv
R
2
L Bvd
3. H:
R
2
L B 2d
4. E:
R
L2 B 2 v
5. B:
R
2. G:

6. J: L2 B 2 d
7. A:

L2 B 2 vd
correct
R

8. I: L2 B 2 vd
9. C: ILBd
10. F: ILB
Explanation:
A is the correct choice. First note that as
the circuit moves to the right, the magnetic
flux through through the circuit decreases.
Consequently, a current I must flow counterclockwise to oppose this change in flux; on the
vertical segment of wire inside the magnetic
field region, this current is acted upon by the
magnetic field, producing a force ILB to the
left.
We are told that the circuit moves at a constant speed v; to move at a constant speed,
net acceleration must be 0, so the applied
force must be equal and opposite to the magnetic force on the current: Fext = ILB. Since
only half the circuit is in the field region, the
total work done to remove the circuit from the
field must be
W = F d = ILBd
Then, using I =

E
and applying Faradays
R

law,
W = ILBd =

ELBd
R



dB LBd

W =
dt R

Version 042 midterm 03 chiu (56565)


W =

d
LBd
(LxB)
dt
R

Let : m = 53 kg ,
M = 820 g = 0.82 kg ,
= 96 cm = 0.96 m ,
B = 640 mT , and
R = 96 m = 0.096 .

L2 B 2 d dx
W =
R dt
L2 B 2 vd
W =
R
where the x introduced in line 3 refers to
the length of the circuit that remains in the
field region.
017 10.0 points
A bar of negligible resistance and mass of
53 kg in the figure is pulled horizontally across
frictionless parallel rails, also of negligible resistance, by a massless string that passes over
an ideal pulley and is attached to a suspended
mass of 820 g. The uniform magnetic field
has a magnitude of 640 mT, and the distance
between the rails is 96 cm. The rails are connected at one end by a load resistor of 96 m.
53 kg

96 cm

640 mT

m
96

820 g

640 mT

a
What is the magnitude of the terminal velocity (i.e., the eventual steady-state speed
v ) reached by the bar? The acceleration of
gravity is 9.8 m/s2 .
1. 3.38626
2. 0.720324
3. 11.6742
4. 20.6214
5. 73.4963
6. 5.20951
7. 2.4138
8. 2.04366
9. 1.97743
10. 1.48119

T
a

Fg

Fm

~ g = M ~g
F
~ net
F

~ m = I ~ B
~
F
~g F
~m
= (M + m) ~a = F
d B
dt
~
~
B = B A

E =IR=

E = Bv.

It follows from Lenzs law that the magnetic force opposes the motion of the bar.
When the wire acquires steady-state speed,
the gravitational force Fg is counter-balanced
by the magnetic force Fm .
Fg = M g = Fm = I B
(1)
Mg
I=
.
(2)
B
To find the induced current, we use Ohms law
d
and substitute in the induced emf, E =
dt
|E|
1 d
I=
=
.
(3)
R
R dt
Note: We have ignored the minus sign from
the induced emf E because we will eventually evaluate the magnitude of the terminal
velocity. The flux is = B A , so

Correct answer: 2.04366 m/s.


Explanation:

640 mT

12

|E| =

dA
d
=B
= Bv,
dt
dt

and

(4)

Version 042 midterm 03 chiu (56565)


Bv
.
(5)
R
Using Eqs. 2 and 5 and noting that v is the
terminal velocity v
I=

B v
Mg
=
.
B
R

(6)

Solving for the magnitude of the terminal


velocity v
M gR
(7)
2 B 2
(0.82 kg) (9.8 m/s2 ) (0.096 )
=
(0.96 m)2 (640 mT)2

v =

= 2.04366 m/s .

keywords:

13

You might also like